Ans Choice B
Hi, The stimulus says that the ocean contains a more proportion of O-16 to O18 than sea water....
ankita96 on August 11, 2020
  • June 1991 LSAT
  • SEC3
  • Q11
1
Reply
Ans Choice E
Hi, I read the previous thread. However I am still not sure how E is wrong. Thanks in advance!
ankita96 on August 10, 2020
  • June 1991 LSAT
  • SEC1
  • Q14
3
Replies
explanation
why isn't is the correct that hodges couldn't join in 1964?
RS1 on August 9, 2020
  • June 1991 LSAT
  • SEC2
  • Q15
1
Reply
Explain why D is correct and C is incorrect
Can someone please explain why C is incorrect and D is correct? I narrowed it down to those 2 but...
RayJ on August 4, 2020
  • June 1991 LSAT
  • SEC1
  • Q4
1
Reply
Q22
Why can't E be true? by looking at option 2 of the hypothetical it seems it is possible that answ...
jam0086@mix.wvu.edu on July 23, 2020
  • June 1991 LSAT
  • SEC2
  • Q22
1
Reply
Why is the answer A?
Hi, Can you please explain why the answer is A and not D?
Steph on July 19, 2020
  • June 1991 LSAT
  • SEC3
  • Q9
4
Replies
Mapping of parallel flaw
Would it be possible to see how we would map out the correct answer and how that logic is the sam...
sydneygrube on July 13, 2020
  • June 1991 LSAT
  • SEC4
  • Q10
3
Replies
Please explain C
Can you please explain C. There have been many requests
fable on July 13, 2020
  • June 1991 LSAT
  • SEC1
  • Q24
1
Reply
I believe I am just requesting the same as ever...
Can you please review why the answer is C?
hspring on July 8, 2020
  • June 1991 LSAT
  • SEC1
  • Q24
2
Replies
Stimulus and Answer Explanation
Hi LSAT Max, I'm having a hard time figuring out this one. Could you explain how to get to th...
Julie-V on July 4, 2020
  • June 1991 LSAT
  • SEC4
  • Q10
5
Replies
Please help
Could you please explain how we can definitively arrive at D as the answer? The stimulus seems a...
yckim2180 on June 30, 2020
  • June 1991 LSAT
  • SEC4
  • Q6
1
Reply
B
How is B relevant?
LillCarr on June 28, 2020
  • June 1991 LSAT
  • SEC4
  • Q11
3
Replies
E?
Didn't the passage say that the amount of oil was equally "extractable" today than from ten years...
LillCarr on June 28, 2020
  • June 1991 LSAT
  • SEC3
  • Q18
1
Reply
Did not know where to start
Can you please break down this question??
LillCarr on June 28, 2020
  • June 1991 LSAT
  • SEC3
  • Q7
4
Replies
B
why is B wrong ?? help
Maria-Marin on June 27, 2020
  • June 1991 LSAT
  • SEC4
  • Q4
1
Reply
D
I chose A and I dont understand why D is right and A is wrong, could someone please help
Maria-Marin on June 27, 2020
  • June 1991 LSAT
  • SEC4
  • Q20
1
Reply
Question
I narrowed down the answer choices to D and E but I am not sure how to differentiate between the ...
claire_crites on June 12, 2020
  • June 1991 LSAT
  • SEC4
  • Q21
3
Replies
Help!
Why can’t A be the answer?
ivandaquial on June 9, 2020
  • June 1991 LSAT
  • SEC3
  • Q17
1
Reply
Why B, Not C?
After reviewing the question, I guess I see why B could be the answer because he must assume that...
avif on May 10, 2020
  • June 1991 LSAT
  • SEC4
  • Q5
2
Replies
Answer Explanation
Hi LSAT Max, I was able to choose (E), but I wasn't too confident about it. If you could prov...
Julie-V on May 7, 2020
  • June 1991 LSAT
  • SEC4
  • Q18
2
Replies